Q8

 
vswamy
Thanks Received: 0
Forum Guests
 
Posts: 9
Joined: December 23rd, 2009
 
 
 

passage 1, Question 8

by vswamy Sat Jun 19, 2010 1:05 pm

The question asks about which quotation from the answer choices given "exemplifies" the atttitude of lines 5-9. My answer was E but the answer is C. Could you explain why?
User avatar
 
ManhattanPrepLSAT1
Thanks Received: 1909
Atticus Finch
Atticus Finch
 
Posts: 2851
Joined: October 07th, 2009
 
This post thanked 2 times.
 
 

Re: passage 1, Question 8

by ManhattanPrepLSAT1 Sat Jun 19, 2010 10:01 pm

We're asked to find a quotation that best exemplifies the "familiar" attitude mentioned in lines 5-9. There the passage states that "enjoyable novels are held to be somehow slightly low-brow, and a novel is not considered true literature unless it is a tiny bit dull."

There are two claims here, and we're looking for an answer choice that matches with either one. We can find an answer choice that says enjoyable novels are low-brow or an answer choice that says that a novel is not considered true literature unless it is a tiny bit dull.

(A) has it all wrong. Truly great literature should not be fantastical and whimsical but rather dull.
(B) has it all wrong. Truly great literature should not be humorous but rather dull.
(C) is the closest match to the statement in lines 5-9. If a work is dull, it does not divert (entertain) the reader, but rather places demands on the reader to work through it.
(D) has nothing to do with either enjoyable literature or truly great literature.
(E) misses the mark by focusing on the requirements of the genre, rather than the experience of the reader.

Does this help clear things up? If you need any further explanation, please feel free to let me know!
 
lhermary
Thanks Received: 10
Atticus Finch
Atticus Finch
 
Posts: 160
Joined: April 09th, 2011
 
 
 

Re: Q8

by lhermary Tue Jan 10, 2012 3:17 pm

Can you go into more detail as why E is wrong? It seems to be the most logical answer.
User avatar
 
ManhattanPrepLSAT1
Thanks Received: 1909
Atticus Finch
Atticus Finch
 
Posts: 2851
Joined: October 07th, 2009
 
 
 

Re: Q8

by ManhattanPrepLSAT1 Sat Jan 14, 2012 12:16 am

The problem with answer choice (E) is that it's too much related to the main thrust of the passage and not enough to the attitude described in lines 5-9. There, a false opposition is described and we're looking for an answer choice that harkens back to that attitude.

enjoyable (easy to read) fiction = low brow
true literature = tiny bit dull (demanding of the reader)

Try to choose an answer choice on questions like this that are closer (literally the text you use to justify your answer choice should come from right around the quote in question) to the line reference.

Answer choice (E) also seems to get it wrong. The author doesn't seem to suggest that all the author needs to do to produce great literature is to follow the genre's requirements, but seems to suggest that good literature places demands on the reader.

Hope that helps!
 
wguwguwgu
Thanks Received: 5
Jackie Chiles
Jackie Chiles
 
Posts: 39
Joined: January 17th, 2012
 
 
 

Re: Q8

by wguwguwgu Sat Apr 07, 2012 1:03 pm

I picked B because of the wording "happens to be". Doesn't this phrase, in contrast to A which says "qualify them as", preclude a usual association being "humorous" (enjoyable) and "great"?

In HINDSIGHT (sigh), I see that the sentence in the passage is phrases in a way that "interesting" and "good" is mutually excluding, so even "happens to be" can't be right. Is this really the reason that B is wrong? that's so annoying, mean, malicious --- they give you the contrast between B and A that prompts you to pick the better one, but wait, B is also wrong!
User avatar
 
ManhattanPrepLSAT1
Thanks Received: 1909
Atticus Finch
Atticus Finch
 
Posts: 2851
Joined: October 07th, 2009
 
This post thanked 1 time.
 
 

Re: Q8

by ManhattanPrepLSAT1 Mon Apr 09, 2012 7:37 pm

wguwguwgu Wrote:Is this really the reason that B is wrong?

I think you're too caught up in some of the peripheral language. The reason that answer choice (B) is wrong is that it doesn't place any demands on the reader. The passage says that great works are a bit dull/demanding, whereas books that are enjoyable are lowbrow.

Answer choice (B) does not reflect this "familiar" attitude. Answer choice (C) however does.

Does that answer your question?
 
einuoa
Thanks Received: 11
Elle Woods
Elle Woods
 
Posts: 51
Joined: January 05th, 2014
 
 
 

Re: Q8

by einuoa Sun Jul 27, 2014 5:46 pm

mattsherman Wrote:
wguwguwgu Wrote:Is this really the reason that B is wrong?

I think you're too caught up in some of the peripheral language. The reason that answer choice (B) is wrong is that it doesn't place any demands on the reader. The passage says that great works are a bit dull/demanding, whereas books that are enjoyable are lowbrow.

Answer choice (B) does not reflect this "familiar" attitude. Answer choice (C) however does.

Does that answer your question?


I was hesitating on C, but I couldn't make the inference that "place demands upon its readers" means that it is dull and that "divert them" means that it's exciting, or what not. Could you explain this more in detail?
User avatar
 
maryadkins
Thanks Received: 640
Atticus Finch
Atticus Finch
 
Posts: 1261
Joined: March 23rd, 2011
 
 
 

Re: Q8

by maryadkins Thu Jul 31, 2014 8:13 am

It's not a perfect match, it's just the closest.

Placing demands is standing in for being dull so presumably difficult to read at moments. Divert means entertain by being "enjoyable" but in a way that doesn't challenge. Again, not perfect synonyms, but the closest of the 5 which is what you're always going for in Reading Comp.